« first day (31 days earlier)      last day (543 days later) » 
00:00 - 14:0014:00 - 00:00

12:04 AM
0
Q: Use Green’s Theorem to evaluate this problem?

user36041Use Green’s Theorem to evaluate \int_cF⋅dr, if F(x,y)=<{Sqrtx}+y^3, x^2+{Sqrty}> and C consists of the arc of the curve sinyx= from (0, 0) to (π, 0)and the line segment from (π, 0) to (0, 0). Trying to get the hang of the coding, so forgive if their are multiple edits. There is a photo tagged t...

This site uses MathJax formatting of formulas. More tips here. (from a bot)Normal Human 21 secs ago
0
Q: Considering the coordinates x = sinh u * sin v, y = sinh u * cos v, how do I show that these coordinates are orthogonal?

puzklwI know that orthogonal means meeting at right angles, but I'm unsure about how to go about computing it. Also, how do I find grad(f) for these coordinates?

 
0
Q: Too many values in [params]?

Andrew GrimmShould the tag params be split into different tags? It isn't the name of an abstract concept - that's be parameters. In Rails, params is a specific term (not quite a keyword, but close) - it refers to parameters available in ActionController. But it's also used by other languages and/or framewo...

 
0
Q: Can we, in a certain way, quantify the measure of non-differentiability of functions that are continuous everywhere but differentiable nowhere?

Ante PaladinI am not sure how to ask this question because it seems to me that my thoughts on this topic are not clear enough, but I will give it a try. What, really, do I want to know? Well, I would like to know is there any "measure" on how "far" is such a function from being differentiable at some point...

Consider replacing (analysis) with a more specific tag for the relevant branch of analysis. (from a bot)Normal Human 21 secs ago
0
Q: Line Interesection Problem

Archios The Problem is: Find the equations of the two straight lines, each of which cuts all four of the lines x = 1, y = 0; y = 1, z = 0; z = 1, x = 0; x = y = 6z I parameterized the four lines as: a(q)=(1,0,q), b(r)=(r,1,0), c(s)=(0,s,1), and d(t)=(6t,6t,t). I've constructed two solution li...

Short title. Title contains problem. Line Interesection Problem
0
Q: 4x4 Matrix Multiplication Equation

john johnI have three 4x4 matrices in this equation : A=XB And I want to find the matrix X. What actions should I do ?

0
Q: Current Research Using Sieve Methods

HoopaUI've been learning various basics of Sieve Methods in Analytic Number Theory, and I'm wondering what are some uses of these methods in current research? Not famous, unsolved problems, but areas of research currently underway? I understand sieve methods' uses in the context of enumerating primes...

Welcome to Math.SE, HoopaU. Consider replacing (analysis) with a more specific tag for the relevant branch of analysis. (autocomment)Normal Human 21 secs ago
0
Q: Discrete Mathematics Troubling Question - Please help ):

Johnnyhttp://imgur.com/a/obzIX The question is that imgur link. Basically, I have no clue how to go about this question at all. Any help would be appreciated. Thank you in advance ):

Welcome to Math.SE, Johnny. Words such as question, please, help are uninformative in titles. Please edit the title so that it better describes the specifics of your question. Do not hesitate to make it longer or include a formula if needed. More tips here. (from a bot)Normal Human 21 secs ago
0
Q: An irreducaible singular cubic hypersurface in $P^n$ is rational.

VincentHow can I show an irreducaible singular cubic hypersurface in $P^n$ is rational. Such question always seems very bizarre for me. Totally no idea. So can any one give me some help?

 
12:39 AM
0
Q: fifth grade word problem

amandaTori has a wooden block shaped like a rectangular prism the area of the base of the block is 96 square centimeters the volume of the block is 1440 cubic centimeters what is the height in centimeters of Tori's wooden block

Short title. Title contains problem. fifth grade word problem
0
Q: How can I prove that there is a bijective function?

AlbertLet A be a nonempty set. Prove that there is a bijective function F : {Equivalence relations on A} → {Partitions of A}. I am completely lost on where to proceed with this question.

Short question. Tagged proof-explanation. How can I prove that there is a bijective function?
0
Q: Is there a case in which Green's theorem will not work, but Stokes theorem will?

user59799I "know" that you can use Green's theorem to integrate over a region bounded by a closed loop. But this is only if the vector field has continuous first partial derivatives inside the region. So I thought that Stokes theorem allowed us to avoid the discontinuity in the 2D region by "going over...

Questions tend to get more attention when they have a tag for a broad area of mathematics relevant to the question. Some of these tags might fit. (from a bot)Normal Human 21 secs ago
 
12:56 AM
0
Q: Why is ${n \choose k} ≥ 1$?

mavaviljWhy is $${n \choose k} ≥ 1$$ I've looked at the expansion of the binomial coefficient, but can't see why the nominator is larger or equal to the denominator.

Short title. Short question. Why is ${n \choose k} ≥ 1$?
0
Q: Test Hypothesis

Bhavik P.I need help solving the following problem. I exhausted many options however I fell short. Any help is appreciated. Creative Good, a New York consulting firm, claimed that 35% of shoppers fail in their attempt to purchase merchandise on-line because Web sites are too complex. A random sample of 6...

Welcome to Math.SE, Bhavik P.. Questions tend to get more attention when they have a tag for a broad area of mathematics relevant to the question. Some of these tags might fit. (autocomment)Normal Human 21 secs ago
 
1:11 AM
0
Q: What exactly is this equation?

AlexThank you for assistance, I'm just having issues remembering what this is called? For example, the equation would go like this |x+1| = 4 What is this type of equation called, with the two | | ? Thanks!

Consider adding a tag for a broader subject area to which the question belongs. Some of these tags might fit. (from a bot)Normal Human 21 secs ago
0
Q: help with modulus prove that deals with induction

Syed NaqiI have this exercise that I was doing and came to this question that I don't know how to prove so here is the question: It is question 5a) and 5b) from this image: enter image description here Recall the definition for a ≡n b and that for a ≡n b, we can write a = nq1 + r and b = nq2 + r where ...

Welcome to Math.SE, Syed Naqi. Words such as help do not add information to titles. Please edit the title so that it better describes the specifics of your question. Do not hesitate to make it longer or include a formula if needed. This site uses MathJax formatting of formulas. More tips here. (autocomment)Normal Human 21 secs ago
0
Q: Dividing by complex numbers

JoceI came across a problem: -16/4i. Everytime I put it into a calculator, it comes out as 4i, but when I try to solve it is -4i, because of the negative one in front of the 16.

Short title. Short question. Dividing by complex numbers
0
Q: discrete math binomial theorem problem: computing the sum

rochellethis is the exact problem i know this question involves the binomial theorem but i am not sure what to do

Title contains problem. Short question. discrete math binomial theorem problem: computing the sum
0
Q: Giving recursive deifnition

JoshI need to give the recursive function of 3n^2. I'm pretty sure the base case needs to be 3(0)^2 = 0, but I don't know where to go from there. Any help is appreciated.

Short title. Short question. Giving recursive deifnition
0
Q: Degree one branched cover is a homeomorphism

Harry BarberSuppose that $f:X \to Y$ is a branched cover of Riemann surfaces and a covering map of degree one outside of the ramification points. Then is $f$ a homeomorphism?

 
1:30 AM
0
Q: What do I do if I can't figure out why OP wrote the wrong code they wrote?

KevinOK, that probably sounds weird, so let me explain. Wrong code is good. It helps us understand the mistake or misunderstanding OP is making. But sometimes, I can't figure out how OP thought the code would work in the first place. Sometimes I get the sense that they are just throwing random cod...

 
0
Q: If $n$ even integers are selected, then there must be a pair who share the same remainder under integer division by 10...

bankeyLet $P(n)$ denote "If $n$ even integers are selected, then there must be a pair who share the same remainder under integer division by 10." Let $k$ denote the smallest value for $n$ for which $P(n)$ is true. What is $k$? Prove that $P(k)$ is true and that $P(k - 1)$ is false. I'm not sure how to...

0
Q: Need a start from 1st grade Math

Bia ParcoGuide me learning Maths from very basic level,concepts and basic numbers, lines and addition, division, multiples,

0
Q: What are some fields isomorphic to $\mathbb{R}^2$ (as a group)?

sacchObviously, $\mathbb{C}$, the field of complex numbers is. But what other fields are? Is it possible to construct some degree of analysis on them?(just luke complex analysis on $\mathbb{C}$)

0
Q: Prove that there exists only 1 prime number of the form $p^2−1$ where $p≥2$ is an integer.

Noah Dengby factoring $p^2−1$, we have $(p+1)(p-1)$. I know that p=2 which gives 3 is the only solution, however how do I prove that p=2 is the only integer which gives a prime?

0
Q: How to determine if a function is a location-scale family

Lysla RiLet $X$ have pdf $f(x)=\frac{1}{\beta}e^{-(x-\alpha)/\beta}$, $x>\alpha$ How to determine if the function f(x) is a location-scale family

 
1:48 AM
0
Q: Opposing `vote to close`

martinI see an almost duplicate here, but the emphasis seems to be on retracting. There is a vote to close option, but (in the interests of democracy ;)) is there an option for vote against close? Should there be one? Has there been any development on this? If so, what is the expected timescale?

 
0
Q: The trigonometric expression 1/2sin(x) is equivalent to

melanieThe trigonometric expression 1/2sin(x) is equivalent to: the answer says sin(x/2)cos(x/2) which I do not understand. Do you not factor out the 1/2 so there would be a one half at the front?

This site uses MathJax formatting of formulas. More tips here. (from a bot)Normal Human 20 secs ago
0
Q: How to put a function in a function in matlab and plot it?

GaussNow I am new to matlab, and now the basic syntax, however I am not sure how to write a function to loop for instance the following recurrence equation: $y(n+1)=\exp^(-y(n))$ .. $\text{where } y(1) = \exp^(-y(0))$ and $y(0) = x$

0
Q: Maximum value of $n$ such that the distances are equal

User 1upon0For a natural number $j$, let $Q_j$ denote the point $(0,j)$ in the coordinate plane. Find the maximum value of $n$ such that there are $n$ points $P_1,P_2 \ldots P_n$(not necessarily different) with integer coordinates all lying on a line parallel to the x axis such that $P_1Q_1=P_2Q_2=\ldots P_...

0
Q: find a recurrence relation for the sequence

rochellethe problem I know that part a is a1=2 because it can be x or y and that a2=3 bc it can be xy or xx or yx. But i do not know how to find part b, the recurrence relation.

0
Q: Is the exponential distribution of a lognormal distribution 1?

Simone BlondAnd vice versa? Can the lognormal distribution be represented exponentialy?

0
Q: How to find the average hop count in a 64 node hypercube?

ikisI was wondering if anyone can help me in figuring out average hop count in a 64 node hypercube.

0
Q: A integration for a probability problem

Lysla RiLet $X$ and $Y$ be independent uniform(0,1) random variables. Find $P(XY\leq t)$ My question is when $t\leq 1$, why does it can be seperated to $P(XY\leq 1)= \int_0^t\int_0^1 dydx+\int_t^1\int_0^{t/y}dydx$

 
2:18 AM
1
Q: Stack overflow app coment issue?

Ruchir BaroniaI have downloaded the Stack exchange app on my phone. Now, if I start typing up a comment, and turn the screen mid-way through my comment: (http://www.solec.org/wp-content/uploads/2015/02/icon-horizontal.png) Then, the comment disappears (which is very frustrating after having typed 300 chara...

 
0
Q: Probability of a Sum of Random Variables

JustinI know that xi are distributed poisson with parameter lambda. Can anyone explain why this holds? Thanks[1]

0
Q: multiplication of consecutive prime numbers in the form $4k +3$

Noah DengHow can I prove that prime numbers beginning with 2, multiplied with the next consecutive prime plus 1, 2x3x5x7....+1, will give the form $4K+3$

0
Q: Alpha and Beta question [Addmath, quadratic equations]

Bhalemtina ParkPlease help. Question: Addmath (Quadratic Equations) Given α and β are the roots of the quadratic equation 2x2 - 6x + 5 = 0. Form an quadratic equation with the roots α + 1 and β + 1.

Welcome to Math.SE, Bhalemtina Park. Words such as question do not add information to titles. Please edit the title so that it better describes the specifics of your question. Do not hesitate to make it longer or include a formula if needed. Consider adding a tag for a broader subject area to which the question belongs. Some of these tags might fit. More tips here. (from a bot)Normal Human 21 secs ago
0
Q: What is wrong with my Laplace Transform?

user1833028I need to find the Laplace transform of sine(t), and it is proving rather difficult. I integrate by parts, then integrate by parts again so that the original integral is on both sides of the equation, then I get it wrong. $ f\left(x\right) = \left\{ \begin{array}{lr} \sin t & : 0 \l...

0
Q: With a 45-gallon and a 124-gallon jug with no marketings (but known capacities), how do you measure exactly 1 gallon of water?

Ilia LabkovskyI could imagine filling up the 124, then filling the 45 from it, so there is 124-45=79 left in the 124, then again so there is 37. Then I'm lost!

0
Q: Olympiad problem algebra inequality

MichaelI'm having trouble solving the following inequality problem: If $n$ is positive integer greater than $1$, and $x>y>1$, then show that: $\frac{x^{n+1}-1}{x(x^{n-1}-1)} > \frac{y^{n+1}-1}{y(y^{n-1}-1)}$ Any hints? Thanks.

Tag (contest-math) should not be the only tag a question has. Please add a tag for a subject area to which the question belongs. (autocomment)Normal Human 21 secs ago
0
Q: Supose $f'(x) \ge M \gt 0$ for every $x \in [0,1]$...

I.PadillaSupose $f'(x)\ge M\gt 0$ for every $x \in [0,1]$. Prove that there exists an interval of length $\frac 14$ where $$| f(x) |\ge \frac M4$$

Short question. [Supose $f'(x) \ge M \gt 0$ for every $x \in [0,1]$...](math.stackexchange.com/q/1557545)
0
Q: using mayer-vietoris to show X(M) = X(U)+X(V)-X(U intersect V)

Rob LiptonLet M be a manifold, and U,V open sub-manifolds in M. How would one use the Mayer-Vietoris theorem to show that X(M) = X(U)+X(V)-X(U intersect V), where X() is the Euler characteristic?

0
Q: Solving Wave equation using D'Alembert Formula

Joy GardeniaLet u satisfy the wave equation u_tt = u_xx, where x ∈ R, t > 0. u(0, x) = f(x), u_t(0, x) = g(x), x ∈ R, where f(x) = sin πx if 2 ≤ x ≤ 3, f(x) = 0 otherwise, and g(x) = 1, if 4 < x < 5, g(x) = −2 if 6 < x < 6.5, g(x) = 0 otherwise. Find all values of t for which u(t, 1)≠ 0. I had t...

This site uses MathJax formatting of formulas. More tips here. (from a bot)Normal Human 21 secs ago
0
Q: 10 people in a room, 2 cannot be near each other, combination problem.

Richard10 people in a room, 2 cannot be near each other. Determine the number of ways the people can be placed in a line without 2 of them being next to each other.

0
Q: Does 3-SAT reduce to 3-CNF-SAT?

John RaeslyI know that SAT goes to 3-SAT and SAT is reducible to CNF-SAT and CNF-SAT is reducible to 3-CNF-SAT but is 3-SAT reducible to 3-CNF-SAT? They are not the same thing though right because cnf makes it conjunctive normal form which is unique in itself.

Title contains sat, sat. Does 3-SAT reduce to 3-CNF-SAT?
0
Q: Airport Networking Flow Problem

RodI've been studying some introductory network optimization, and I noticed an interesting problem about flight networks. Suppose there are four locations $A$, $B$, $C$, and $D$ hold. One can fly from $A$ to $B$, $A$ to $C$, $B$ to $A$, $B$ to $D$, $C$ to $B$, $C$ to $D$, $D$ to $A$, and $D$ to $C.$...

0
Q: Gamma Distribution sample question

QualityHello I am wondering about how to approach the following question; Suppose we are told that the weight of each gum ball ( in centigram) is given by the gamma distribution function, with $\alpha=25$ and $\beta=2$. We are wanting to know the probability that 100 gum balls will go over the limit o...

Consider adding a tag for a broader subject area to which the question belongs. Some of these tags might fit. (autocomment)Normal Human 21 secs ago
 
2:56 AM
0
Q: Train problem, probability

shoestringfriesIf one commuter train comes every $15$ minutes and another comes every $40$ minutes, what is the average amount of time one would have to wait before getting on a train? Suppose that they are not synchronized. The answer by the way is $6.5$ minutes. I don't see how the trains are not synchroni...

Short title. Title contains problem. Train problem, probability
 
3:07 AM
0
Q: Identity of $8sin^2(t)cos^2(t)$

acupajoeI know this probably has a simple answer, but I am having trouble understanding the steps to find the identity for this problem. My guess is some kind of $8sin^2(x)cos^2(x) = 2sin^2(2x)$ The closest Identity I can find is: $sin(x)cos(y) = 1/2[sin(x+y) + sin(x-y)]$ Which would give $1/2[sin(...

Short title. Tagged algebraic-identities. Identity of $8sin^2(t)cos^2(t)$
 
0
Q: Why more than a thousand views for my 'closed' question?

JayI had asked this question some time back. What I intended to know was understood perfectly by the person who has answered it. Since then, the question has been viewed a thousand times earning me a 'popular' badge. It shows that lot of people had that question in mind. Surprisingly, I have only on...

 
0
Q: ∀a∈N,∀b∈N,∀k∈N,a≡nb→ak≡nbk

danny13Prove that for $n\in \mathbb N, n\geq 2$: $\forall a\in \mathbb N, \forall b \in\mathbb N, \forall k\in \mathbb N, a\equiv_n b \rightarrow a^k \equiv_n b^k$. the definition for $a \equiv_n b$ and that for $a \equiv_n b$, we can write $a = nq_1 + r$ and $b = nq_2 + r$ where the remainders $r$ ...

Short title. Tagged proof-writing. ∀a∈N,∀b∈N,∀k∈N,a≡nb→ak≡nbk
 
3:24 AM
0
Q: Show that in a group of seventeen people, there exists a trio who are either three mutual friends, three mutual enemies, or three mutual strangers.

bankeySuppose that in a group of people that any two people are either friends, enemies of strangers. Show that in a group of seventeen people, there exists a trio who are either three mutual friends, three mutual enemies, or three mutual strangers. I know I need to use the Generalized Pigeonhole Prin...

0
Q: Integral of delta function and derivative of delta function

Victor V AlbertCan anyone rigorously prove this? $$\int dx\delta\left(x-\alpha\right)\delta^{\prime}\left(x-\beta\right)=\delta^{\prime}\left(\alpha-\beta\right)$$.

0
Q: recursive sequence $x_{n+1}=\frac{1}{x_n}$

babylon$x_{n+1}=\frac{1}{x_n}$, $x_1>1$ Find if the function converges and diverges and then prove it. If we try and find the limit we get 1 or -1. 1 does not work because then $x_1$ and $x_n$ contradict each other. -1 works for showing its the lowest bound. I'm not sure about this proof for proving i...

 
3:39 AM
0
Q: To provide an example and illustrate one valid use of the Accept / Reject Algorithm

Spencer IrelandI was wondering what example can I come up with to illustrate the valid use of Accept / Reject Algorithm.

0
Q: Advice needed to understand a logic problem

Alexy VincenzoI have difficulty understanding the solution to this problem and would appreciate if someone can explain why my solution is wrong: You meet a person who either always lies or always tells the truth. He flips a standard coin and makes a the following statement: The toss is head if and only if I a...

0
Q: Help compute an entropy

TonyCompute the entropy of the density function $\frac{b}{{\pi ({b^2} + {x^2})}}$. I think the entropy of a density function $f(x)$ is given by $H = -\int f(x) \ln f(x) ~dx$ My calculation is $H = - \int_{ - \infty }^{ + \infty } {\frac{b}{{\pi ({b^2} + {x^2})}}(\log \frac{b}{\pi } - \log ({b^2} +...

Words such as help do not add information to titles. Please edit the title so that it better describes the specifics of your question. Do not hesitate to make it longer or include a formula if needed. More tips here. (from a bot)Normal Human 21 secs ago
0
Q: Proof that 3Partition is NPC

imabelianWe have PARTITION3 = {(S)| S is a multi-set of positive integers that can be partitioned into 3 sets where the sum of each set is equal} I want to prove that this is NP-complete. To show that it is in NP is pretty simple. You just guess 3 partitions nondeterministically and check if the sum is...

Consider adding a tag for a broader subject area to which the question belongs. Some of these tags might fit. (autocomment)Normal Human 21 secs ago
0
Q: Transform the QM radial equation to spherical Bessel equation

Jane M.I'm currently learning about spherical potentials in the context of quantum mechanics (ex. hydrogen and hydrogen-like systems) and am trying to work through the problem of a generic spherical potential well. I'm comfortable with the separation of variables and how the radial equation is obtained...

Tagged differential-equations but mentions "partial". Transform the QM radial equation to spherical Bessel equation
0
Q: Integration using residue

plumsauce5Show that \int_{0}^{\infty} 1/(1+x^n)dx=(\pi /n)/sin(\pi /n) where n is a positive integer. He wants us to perform a contour integration on a "pie slice" containing a pole. We're supposed to find the singularities and then find the residue to solve it. I'm not ever sure how to approach it becau...

0
Q: Control Systems Theory--State Equation

bh724I am really just unsure of how to start this problem, any kind of help/hint would be much appreciated. Continuos Stirred Tank Reactor

 
3:57 AM
1
Q: CSS Canonical for "pseudo elements need a content property"

cimmanonThis is a somewhat common problem that crops up every now and then where the asker doesn't realize they need to set the content property on pseudo elements in order to bring them to life. Unfortunately, I can't seem to find a definitive canonical to use when closing them as duplicates. Here's t...

0
Q: What is the difference between declined, disputed and aged away?

Mohit ShrivastavaI am not able to understand the flagging history. So when I flag a post like Very low Quality and it is declined like declined - a moderator reviewed your flag, but found no evidence to support it does the moderator looked at the non-edited post which was flagged or does the moderator look at t...

 
0
Q: Proof of trignometric identity

user295440Could you help me prove this? I've gotten stuck, need some help.. sin^2Θ + tan^2Θ = sec^2Θ - cos^2Θ Here's what I've done so far: Left Side: sin^2Θ + sin^2Θ/cos^2Θ ((sin^2Θcos^2Θ)+sin^2Θ)/cos^2Θ Right Side: 1/cos^2Θ - cos^2Θ (1-cos^2Θ(cos^2Θ))/cos^2Θ Thanks in advance. Note: I have trie...

Welcome to Math.SE, user295440. This site uses MathJax formatting of formulas. More tips here. (from a bot)Normal Human 21 secs ago
0
Q: Gambling Question

Jonathan YoderI had a question regarding blackjack. In the game, the odds of winning are about 48 percent or something. I think the limit of your profit as the number of games played approaches infinity would be 48 percent. Based on this, the odds of earning money would be zero after playing an infinite amount...

Words such as question do not add information to titles. Please edit the title so that it better describes the specifics of your question. Do not hesitate to make it longer or include a formula if needed. More tips here. (from a bot)Normal Human 21 secs ago
0
Q: Need a smart solution of the Integral

FukuzitaProve that, $\int_0^1\int_0^1 \frac{x^2dxdy}{\sqrt{(1-x^4)(1+y^4)}}=\frac{\pi}{4\sqrt{2}}$

 
0
Q: Rewrite link-only answer completely with extensive quote reformatting, or just write own answer?

Nathan TuggyEvery so often (say, about five times a day) I come across a link-only answer that could probably be rewritten to include a direct quote of most or all of the original. Or, it could be deleted. Hitherto I have been picking between those, depending mostly on the amount of energy I can spare to the...

-2
Q: Whats wrong with my questions?

SyntaXI have these questions posted earlier, I want to know why they are down voted. There are no down vote comments. Here is the details: Java Performance concern in Exception Handling Is there an Api in JSX to ignore if self closing tags like is not closed? Is there anything similar of JPA 2.1 En...

 
4:18 AM
0
Q: Three term inner and outer solution to a boundary layer problem

MattI am unsure how to proceed with the current equation to determine a three-term outer expansion and three-term inner expansion due to the nature of the equation. Equation: $\epsilon \frac{d^2y}{dx^2}+\frac{dy}{dx} = 2x$, $y(0) = 0$, $y(1) = 2$ Whilst I only have lecture notes with equations in...

0
Q: Find the last row of a 3x3 matrix given the eigenvalues

MichelleI was able to get the last row of A by replacing the asterisks with x and y, doing (0-λ)(x-λ)-(x)=0 and plugging in 4 and 7 for λ. I got x=-28 and y=11. However, I tried to do the same for matrix C and could not get the values for the last row. My work resulted in (z-1)=(4z-8)=(9z-27), which does...

0
Q: Show that a function has both positive and negative on the neighborhood of 0

CjugThe function: $$f(x)=4x\sin\left(\frac{1}{x}\right)-2\cos\left(\frac{1}{x}\right)+1 ,\text{ if }x\neq0$$ takes both positive and negative values on any open interval around $x=0$, i.e. on interval (-c,c). There's hint that any such interval contains zeroes of both cos(1/x) and sin(1/x), but I'm ...

0
Q: Problem related matrix and counting

ArupI have a following matrix related problem: Let $F$ be a $n \times n$ discrete Fourier matrix defined as $F_{j,k} = \frac{1}{\sqrt{n}}exp{(\frac{i 2\pi jk}{n})}$, for $0 \leq j,k \leq n$, where $i^2 =-1$. For $x \in C^n$, let $l(x)$ denote the number of non-zero components of $x.$ Show that for...

Title contains problem. Problem related matrix and counting
0
Q: Is the scalar multiple of an eigenvector also an eigenvector for a particular eigenvalue?

Clark BellI'm working on a problem from my textbook and found that (1/2, 1/2, 1) is an eigenvector for a particular eigenvalue of 4. The textbook solution says that the answer is (1, 1, 2) which is just 2 * (1/2, 1/2, 1). Thanks in advance.

Welcome to Math.SE, Clark Bell. This site uses MathJax formatting of formulas. More tips here. (from a bot)Normal Human 21 secs ago
0
Q: The three Cevian's are concurrent at point T. Show that all six small triangles have equal areas.

Charles It is given that $triangle ART$, $triangle BPT$, and $triangle CQT$ have an area of one. I have no idea how to approach showing that all of the triangles have an equal area of one.

0
Q: Proof that problem is np complete

PanphobiaIf you have Part = {(S)| S is a set that can be partitioned into 3 parts whose sums are equal} Prove that Part is NP Complete. So to show Part is in NP is easy. Just guess 3 partitions non deterministically and check sums. But to show Part can be reduced from Subset Sum is a little tricky. I ...

Tag (proof-explanation) should not be the only tag a question has. Please add a tag for a subject area to which the question belongs. (from a bot)Normal Human 20 secs ago
Title contains problem. Tagged proof-verification. Proof that problem is np complete
 
4:51 AM
0
Q: {E_{TM}}$ is NP-hard

MaryI'm using Sipser's book and after several attempts I still don't quite understand how to show $\overline{E_{TM}}$ is NP-hard.

Short title. Title contains hard. Short question. {E_{TM}}$ is NP-hard
 
5:04 AM
0
Q: On Cantor's argument

TurboWhat axioms of set theory are needed for Cantor's diagonalization argument to work and why? What happens if we do away with some of these axioms (for instance Axiom of Choice)?

Short title. Short question. On Cantor's argument
0
Q: Hypergeometric Distribution Definition

Moon LeeI have a definition of a Hypergeometric distribution as follows: Definition: the Hypergeometric distribution is a discrete probability distribution that describes the probability of $k$ successes in $n$ draws, without replacement, from a finite population of size $N$ that contains exactly $K$...

0
Q: Compute ratios $\frac{XP}{PY}$ and $\frac{BQ}{QC}$ in terms of lengths r=$\overline{AX}$, s=$\overline{XB}$, t=$\overline{AY}$ and u=$\overline{YC}$

CharlesShow that the only way these ratios can be equal is if both ratios equal one, and show that in that case, $\overline{XY}$ is parallel to $\overline{BC}$.

0
Q: Expectation of $n$ Dependent Random Variables

usedbywhoLet $X_1 \sim U[0,1]$ and $X_i \sim U[X_{i - 1}, 1]$, $i = 2, 3,...$. What is the expectation of $X_1 X_2 \cdots X_n$ as $n \rightarrow \infty$? Thank you very much!

0
Q: ddddjfksdjfsdalkfjsdaklfjsdaklf

박기태laplace residual distribuiton 을 갖는 location model 일때, Q2 가 mle라서 \bar{X} 보다 efficient 한데, ARE 를 구해보면 2가 나와서, 보통 2배 efficient합니다. 건 그렇고, theta 에 대한 CI를 만들때, Q2나 \bat{X} 를 가지고 만들수 있는데, Q2는 MLE라서 mle 정규근사에 의해 정규분포에 근거해서 만들 수 있고, \bar{X}는 sample mean이라서 CLT에 의해 역시 정규분포에 근거해서 만들수 있습니다. (1-\...

Welcome to Math.SE, 박기태. This site uses MathJax formatting of formulas. More tips here. (from a bot)Normal Human 21 secs ago
 
5:35 AM
0
Q: Why are comment upvotes prevented after being undone?

Guruprasad RaoWhy can we not upvote a comment a second time? We upvote comment once, then we undo it and then again if we try to upvote, it says You've already undone your vote on this comment; you cannot upvote it again. Of course we get an alert before we undo, saying that we cannot upvote again, fair enoug...

 
0
Q: MHF4U : Proving Trigonometry Identities

Arbaaz ShakeelI was not able to prove these identities.I was wondering if anyone can answer them since I couldn't. b) sin2x-tan2x = -sin2xtan2x c) (cos2x-1)(tan2x+1)=-tan2x d) cos^4x-sin^4x = cos^2x-sin^2x

Welcome to Math.SE, Arbaaz Shakeel. This site uses MathJax formatting of formulas. More tips here. (from a bot)Normal Human 21 secs ago
0
Q: Use Green's function to find a solution to the initial value problem

D_SConsider a differential equation $y'' + y = F(x)$, where $F$ is continuous on $[a,b]$. If $x_0 \in (a,b)$, I'm supposed to show that the solution to the IVP $$y'' + y = F(x), y(x_0) = y_0, y'(x_0) = y_1$$ is $$y(x) = y_0 \cos(x-x_0) + y_1 \sin(x-x_0) + \int_{x_0}^x F(t) \sin(x-t)dt$$ What I did ...

0
Q: Adding medians.

anonymousIf we called the perimeter of some triangle b. Prove that if you added the lengths of any two of its medians (i) It would not be not larger than (3b)/4 (ii) It would be smaller than (3b)/8 This came up in a math competition some time ago and I think that using the triangle inequality will be n...

Short title. Adding medians.
 
6:21 AM
0
Q: Translated transformation help

user295468GIVEN: Trapezoid $\partial R$, in the $xy$-plane with vertices $A, B, C, D$ given by $A = (1,0), B = (2, 0), C = (0, 2), D = (0, 1)$ Let $R$ be the (finite) region of the $xy$-plane enclosed by $\partial R$ and $f: R \to R, f(x,y) = e^\frac{-x+y}{x+y}$ FIND: $\int\limits_Rf(x,y)dxdy$ HINT: Con...

Welcome to Math.SE, user295468. Words such as help are uninformative in titles. Please edit the title so that it better describes the specifics of your question. Do not hesitate to make it longer or include a formula if needed. More tips here. (autocomment)Normal Human 21 secs ago
0
Q: Questions regarding the existence of normal subgroup with finite index

Lewis If $G$ is a group with a subgroup $H$ of finite index $n$, then $G$ has a normal subgroup $N$ whose index in $G$ is finite. I found a proof of the question here: How to prove that if $G$ is a group with a subgroup $H$ of index $n$, then $G$ has a normal subgroup $K\subset H$ whose index in $...

0
Q: just another $\pi$ formula

Eddy KhemiriI've found this $\pi$ formula: $$ \pi =\text{Limit}_{n\to \infty }\underset{k=1}{\overset{n}{4\text{ }\sum }} \frac{2 n^3 (1-2 k)^2 \left((k-1) k-n^2\right)}{\left(k^2+n^2\right)^2\left((k-1)^2+n^2\right)} $$ What is interesting is that the formula has a geometric origin. So it could have bee...

0
Q: Cumulative and inclusive Tax calculation question

SanI need your help in deriving a formula for calulating three taxes with a comination of cumulative and incusive tax. Here are the examples with expected result. Please let me know the formula on how to get the given result. Example1: Total Product Value (V) = 1000 (with inclusive of all 3 taxes ...

Welcome to Math.SE, San. Words such as question are uninformative in titles. Please edit the title so that it better describes the specifics of your question. Do not hesitate to make it longer or include a formula if needed. More tips here. (autocomment)Normal Human 21 secs ago
0
Q: Silly question about complex numbers - if its modulus is < 1, does raising it to higher exponents make it decrease to the real number 0?

Lebron JamesJust playing around with the modulus definition doesn't really confirm that thought... Is it true? If |z| = |x+iy| < 1, is $$\lim_{n\to \infty} z^n = 0 ?$$ Thanks,

0
Q: Green's Function for coupled odes

Soumyajit RoyI want to solve the following coupled ODEs using Green's function. $\begin{eqnarray} m_1x''+k_{12}x+c_{12}x'-k_2y-c_2y'=f_1(\xi)\quad \text{for}\quad a\leq\xi\leq b\\m_2y''+k_{23}y+c_{23}y'-k_2x-c_2x'=f_2(\xi)\quad \text{for}\quad a\leq\xi\leq b\end{eqnarray}$ where, $x=x(\xi)$ and $y=y(\xi)$. ...

Question contains please. Green's Function for coupled odes
0
Q: Help computing a character table

L. T. P. L.Let $G$ be the group of order 20 defined in terms of generators and relations: $$G:=<x,y|x^5=y^4=1\text{ and }yx=x^2y>.$$ Can anyone help me to derive the character table. According to gap the conjugacy classes can be represented by $1, x, y, y^2$, and $y^3$. I can't determine the sizes of these...

Words such as help do not add information to titles. Please edit the title so that it better describes the specifics of your question. Do not hesitate to make it longer or include a formula if needed. More tips here. (autocomment)Normal Human 21 secs ago
0
Q: Conversion of CFG to CNF

technoI have the following question I have answer for the first one as S->C1S C1->C2A C2->a S->aA S->C2A S->a Hope im right?

Short title. Short question. Conversion of CFG to CNF
 
6:52 AM
0
Q: how should I report question or answer posted using different language?

sadaf2605In my review queue I have got this answer, I can't tell which language the answer is but definitely not English, in that case how can I flag or report this?

 
0
Q: Is my proof of this combinatorics problem corect?

Sayantan SantraI found the following question in a paper I was trying to solve: The following figure shows a $3^2 \times 3^2$ grid divided into $3^2$ subgrids of size $3 \times 3$. This grid has $81$ cells, $9$ in each subgrid. Now consider an $n^2 \times n^2$ grid divided into $n^2$ subgrids of size $n \ti...

 
7:22 AM
0
Q: Problem involving the completion of a general metric space

Greg.PaulA {\em completion} of a metric space $(X,d)$ is a metric space $( \widetilde{X}, \widetilde{d} )$ such that $X \subseteq \widetilde{X}$ and such that: (I) $\widetilde{d}$ extends $d$. That is, we have $\widetilde{d} (x_{1},x_{2}) = d(x_{1},x_{2})$ for every $x_{1},x_{2} \in X$. (II) $X$ is d...

0
Q: Solve for a Fourier series problem

AndyWanna_KnowItAllI am learning Fourier series and I have a problem which has me confused and would like to here others take on it. $$F(t)=\begin{cases}v_0 & \ \ 0 \le t\le T\\ 0 & \ \ T\le t \le \ 2T\end{cases}$$ The problem doesn't have any period given. I have solved it for period $2L=2T$ $=>$ $L=T $ ...

Title contains problem. Solve for a Fourier series problem
0
Q: How to prove $∀a ∈ Z, ∀b ∈ Z, ∀c ∈ Z,(a|b ∧ a ∤ c) → a ∤(b + c)$

Noah DengA, B, and C are all integers. B is divisible by A however C is not. How do I prove the sum of B and C is not divisible by A?

0
Q: Is f(x) = ceiling(x) everywhere defined and bijective?

user295480f(x) = ceiling(x) has a domain of set of real numbers and has a range of set of integer values. So f(x) is everywhere defined? Is it a bijective? Why?

 
0
Q: Add screenshot to help page "What should I do when someone answers my question?"

teylynSome people can read, others need pictures. I find a lot of new members don't know how to accept an answer because they don't see or recognize the check mark. Can the help page What should I do when someone answers my question? please be edited to include a screenshot of what the asker will see...

 
7:39 AM
0
Q: If $f:R\to R$ and $g:R\to R$ be functions such that $g(x)$ is onto and $fog(x)$ is injective then prove that $g$ must be injective.

Vinod Kumar PuniaIf $f:R\to R$ and $g:R\to R$ be functions such that $g(x)$ is onto and $fog(x)$ is injective then prove that $g$ must be injective. I dont know how to prove it.I only know that composition of two injective functions is an injective function and composition of two surjective functions is surjec...

0
Q: Poisson distribution?

TingThe number of sick days taken during a year by an employee follows a Poisson distribution with mean 3. Let us observe 5 such employees. Assuming independence, compute the probability that their total number of sick days exceeds 9.

Welcome to Math.SE, Ting. Questions tend to get more attention when they have a tag for a broad area of mathematics relevant to the question. Some of these tags might fit. (autocomment)Normal Human 20 secs ago
0
Q: Why integral is equal to zero

user59419I wonder why under assumption that w>>$\frac{1}{T}$ then $\int_{0}^{T} sin(wt)dt$ is zero? Since the integral should be like- $\frac{cos(wt)}{w}$ from $0$ to $T$ and after plugging the valued we will end up with : $\frac{-cos(wT)+1}{w}$

0
Q: Prove $f : A\rightarrow B, g: B\rightarrow C$ , and $g\circ f: A \overset{1-1}{\rightarrow}C$, then $g:B\overset{1-1}{\rightarrow}C$

JabernetI am completely stuck on this, I want to say it's true and do a proof by contrapositive, since if g is not surjective, then $\exists b \in B $ such that for $c \in C, f(b)\neq f(c)$, but I'm not sure where to go with this. Thanks!

0
Q: Likelihood model help

Stats123for some reason my mind is drawing a blank when I'm trying to figure out a certain model. For example, if I had a model Xt=Yt+Wt, where t=1,..K, where Y is an exponential(P) distribution and W is a Poisson(Q) distribution, would my likelihood function for l(P,Q;x,y) simply be: l(P,Q;x y) is p...

Welcome to Math.SE, Stats123. Words such as help are uninformative in titles. Please edit the title so that it better describes the specifics of your question. Do not hesitate to make it longer or include a formula if needed. This site uses MathJax formatting of formulas. Questions tend to get more attention when they have a tag for a broad area of mathematics relevant to the question. Some of these tags might fit. More tips here. *(from — Normal Human 21 secs ago
0
Q: thick set syndetic set

James ChanA syndetic set $S$ is a subset of the natural numbers $\mathbb{N}$ or integers $\mathbb{Z}$, having the property of "bounded gaps": that the sizes of the gaps in the sequence of natural numbers is bounded. That is to say, $S$ is a syndetic set, if there exist a positive integer $l$, such that for...

0
Q: For any $x > 0$, show that $\lim_{x\to \infty}\cfrac{x^n}{n!}=0$

ankushaFor any $x > 0$, show that $$\lim_{x\to \infty}\cfrac{x^n}{n!}=0$$

 
7:58 AM
1
Q: Strange Vote Button below my Tags

oberflanschSometimes when I visit/reload the question page, I've got this empty white filed on the right side of the page. If I hover with the mouse over it, a vote button appears, which seems to be somehow missplaced. It is happening in Chrome and Firefox. What is this and can this be removed?

 
0
Q: The ratio of two asymptotically normal distribution

user227924Let $(X,Y)$ be asymptotically normal with their means, variances, and a covariance. Then, I would like to show $X/Y$ is also asymptotically normal. I think there should be some references related to this topic. However, I can't find any documents about ratio distribution in terms of asymptotic pr...

0
Q: Linear algebra, vector space

sharafat salamlet T be a linear operator on vector space v with characteristic polynomial λ^4(λ-4)^5 and minimal polynomial λ(λ-4) then rank of T=?

Short title. Short question. Linear algebra, vector space
0
Q: interchange change two sums

RobinDoes the convergence of the left hand side imply the convergence of the right hand side and the equality of the two sides? Do we need further requirement for the function $f(j,k)$? $$ \sum_{k=0}^{\infty} \sum_{j=0}^{k} f(j,k) = \sum_{j=0}^\infty \sum_{k=j}^\infty f(j,k)$$

 
0
Q: Are math only questions relevant when the math is that of quantum mechanics?

Nick ChapmanIn my brief study of quantum mechanics I have found that math done using operators typically breaks from traditional mathematics. Almost certainly between linear algebra and probably something else there is plenty of mathematics to accurately sum up operators. That being said, does it make sense ...

 
0
Q: proof of $a ≡_n b → a^2 ≡_n b^2$

Noah Deng$a ≡_n b$ can be written as $a = nq_1 + r$ and $b = nq_2 + r$ where the remainders $r$ are the same. Prove that for n bigger or equal to 2 ∀a ∈ N, ∀b ∈ N, $a ≡_n b → a^2 ≡_n b^2$ and therefore $a ≡_n b → a^m ≡_n b^m$ where ∀m ∈ N Where do I begin to prove this? Do I use induction?

Tagged proof-verification. proof of $a ≡_n b → a^2 ≡_n b^2$
0
Q: How to compute $ \lim\limits_{t\rightarrow0}\frac{\int_w^{w+tv}f(y)dy}{t} $?

lanse7pty$w,v\in R$ and $f\in C^1(R)$ . How to compute $$ \lim\limits_{t\rightarrow0}\frac{\int_w^{w+tv}f(y)dy}{t} $$ I feel it should be $f(w)$, but I don't know how to prove it

0
Q: Linear algebra, vector space

sharafat salamIf F:R^n×R^n->R is the function F(x,y)=, where <,> is the standard inner product of R^n and A is a n×n real matrix.Here D denotes the total derivative.which of the following statements are correct? (A)DF(x,y)=+. (B)DF(x,y)=0. (C)DF(x,y) may not exist for some (x,y)€R^n×R^n. (D)DF(x,y) does not ex...

This site uses MathJax formatting of formulas. More tips here. (from a bot)Normal Human 22 secs ago
0
Q: Convergence of 1/n^2

JamgreenI know that $\sum_{n=0}^\infty 1/n$ diverges whereas $\sum_{n=0}^\infty 1/n^2$ converges. Intuitively, I do not see the difference. If $n \to \infty$, the denominators in both fractions will be so big that the fraction approaches zero. So why doesn't both the series converge against zero? I hav...

Short title. Convergence of 1/n^2
 
8:33 AM
0
Q: Exercise to show map from zeros of $\zeta(s)$ to zeros of $C(s)$

danielThis is hopefully a better and clearer version of a deleted question. Let $P(s),C(s),\zeta(s)$ be the prime zeta function, the analogous composite zeta function, and the classical zeta function. I do not know whether it is known that there are infinitely many zeros of C, or whether there exist...

 
8:44 AM
0
Q: Frobenius complement in semidirect product

BungoThis is problem 1.D.4 in Isaacs, Finite Group Theory. I think I have a proof, but it's a rather grungy element-pushing argument (very un-Isaacs in style). My questions are: Is there a cleaner, more "group theoretic" way I can prove this? Unlike most problems in the book so far, I don't really h...

0
Q: Category theory problem? Linear Algebra problem? Pull-back transformations

physicsmajorI'm having a hard time solving this. I'm taking a proof-based undergraduate linear algebra course that has no assigned textbook and this has been making things a little hard since the only resource I have are my notes from lecture -- he's doing things his own way (starts with set theory, then gro...

 
0
Q: Do we need (modern-analysis) tag?

mrfGoing through the suggested edits review queue, I stumbled over a tag-wiki edit to modern-analysis. Since I had never seen that tag before, I had a look, and it was just created with one single question tagged (not by the OP). Do we really need that tag? (Where and when is the differerence betwe...

 
0
Q: Prove or disprove that if f : R → R is differentiable and f' is monotonic, then f ′ must be continuous on R.

DodgeI think to show f' is continuous, I need to show that lim(x->x0) f'(x)=f'(x0) exists. But i don't know how to prove that.

0
Q: Prove $3 * 5 * 7 * 11...prime_n$ = $2k + 1$

Noah Dengit is known that any prime > 2 is odd, however how do I show the combinations of all primes > 2 is indeed odd, or 2K+1? I tried using induction however, what is appropriate for $prime_n$? $3 * 5 * 7 * 11...2k + 1$ = $2k + 1$ ? can we use $2k + 1$ to replace $prime_n$ since we know it will be odd?

0
Q: Expected value for blackjack

Jonathan YoderIn the game of blackjack, the odds of winning each hand are slightly less than 50 percent. As you play an infinite amount of hands, you would always lose money because you would win less than 50 percent of the time. By this rationale, wouldn't your highest odds of winning be if you only played on...

0
Q: Feild Automorphism

kalpeshmpopatlet $G$ denote the group of all the automorphisms of the field $F_{3^{100}}$ that consists of $3^{100}$ elements.Then what is the no of distinct subgroups of G?

Short title. Short question. Feild Automorphism
0
Q: splitting of poisson variables

whoisitthis is some lecture slides from my school. I don't understand why we need to sum up all from m=0 to m=+oo, I think m is a fixed number which represents the number of type 2 event? plz help me! many thanks in advance!

Consider adding a tag for a broader subject area to which the question belongs. Some of these tags might fit. (from a bot)Normal Human 31 secs ago
0
Q: Please, help to identify this algebraic number

fiboI'm trying to find an answer to this question. Let $K(k)$ be the elliptic integral of the first kind and $K'=K(\sqrt{1-k^2})$. According to Abel's theorem (see this link) we know that if $\frac{K'}{K}=\frac{a+b\sqrt{n}}{c+d\sqrt{n}}$ where $a,b,c,d,n$ are integers, then $k$ is the root of an alg...

Words such as please, help do not add information to titles. Please edit the title so that it better describes the specifics of your question. Do not hesitate to make it longer or include a formula if needed. More tips here. (autocomment)Normal Human 22 secs ago
0
Q: reduction of the maximum flow problem and minimum cost flow problem

user269663I'm stack on the following problem. I have to show that the minimum cost max-flow problem can be solved by reducing it to a minimum cost flow problem. Any hint ?

Title contains problem, problem. Short question. reduction of the maximum flow problem and minimum cost flow problem
0
Q: Limit YAW rotation rate

asd7lI've been working on a 3D game for a bit now and simply can't figure out a proper formula for limiting the users YAW rotation rate. I decided to post on this forum with hopes of a possible solution to my issue, so i'm sorry if this is the wrong section. Basically the user ingame should only be ...

0
Q: Struggling to understand argument about number of roots of a polynomial over a field

AuclairOn a previous Cryptography exam I'm working through, there is the following problem: Given $$f(x) = x^{134}+x^{127}+x^{7}+1$$ and the field $\mathbb{F}_{2^{n}}$ where $n=1463$, how many roots does the polynomial have? The polynomial has the form $f(x) = (x^{7}+1)(x^{127}+1)$, this is clear. It'...

 
9:34 AM
-1
Q: Is there, or will there be a way to tag / link / follow "friends" on stackoverflow?

exxeccIn that way i could directly see any new questions / answers / news / updates of my friends / coworkers. Right now i must have the name or userid of my friends / coworkers or dthe direct link of the question to manually get to their account / profile / question. In a company they could link to ...

 
0
Q: Solution to the Laplace's equation

Hamid MohammadI am on my way to solve the hydrogen-atom in 3D, and now I am stucked with the Laplace's equation, where there occurs a "=", which I can't understand.

 
0
Q: Review - "Link is Answer" vs. "Link is Reference"

sebastianbroschI reviewing some cases on First Posts and get the following one: http://stackoverflow.com/review/first-posts/10425701 The questioner post a link with the reference of his following little explanation. The explanation is not the best but in my opinion it is possible to solve the problem. I presse...

 
0
Q: On the base axioms of a matroid

SorropThe base axioms of a matroid state that A collection $B\subseteq 2^E$ is a set of bases of a matroid M(E,I) if and only if the following hold B1: $B\neq \emptyset$ B2: If $B_1,B_2\in B$ and $x\in B_1 - B_2$ then $\exists y\in B_2-B_1$ such that $(B_1 - \lbrace{x\rbrace})\cup \lbrace{y\rbrace}\...

Consider adding a tag for a broader subject area to which the question belongs. Some of these tags might fit. (from a bot)Normal Human 24 secs ago
0
Q: Counting number of funcitons

punjabidon$ f(kx$ $mod p)$ $\equiv$ $k.f(x)$ $mod p$ for some function f defined as $f: \{0,1,....p-1\} \rightarrow \{0,1,....p-1\} $, where p is an odd prime number and k lies in the range $[0,p-1]$ (both p and k are fixed integers). We have to find the number of distinct functions, f, for which this re...

0
Q: Set theory implications

o.fithcheallaighI'm pretty new to the world of fuzzy set theory, and I am trying to understand implications. So, I am wondering if someone help tell me if the following is correct. I am trying to find the minimum of: $$ H \rightarrow \lnot G $$ and any advice or comments would be fantastic. My attempt is:$$ H ...

0
Q: calculation of first moment

adamGI'd like to calculate the following: $$ \int y\partial_{y_2} [(y_1+y_2)p(y)] dy,$$ where $p$ is a probability density and $y=(y_1,y_2,y_3)$ is a vector. I need integration by parts here so I need the integral $$ \int \partial_{y_2} [(y_1+y_2)p(y)] dy.$$ The first moment is the centre of mass...

 
9:52 AM
0
Q: Why Votes on meta.stackoverflow.com dont affect reputation but that is not the case with meta.stackexchange.com

N-JOYJust observed that Votes on meta.stackoverflow.com do not affect reputation but User reputation gets affected by upvotes/downvotes on meta.stackexchange.com What is the specific reason behind this?

 
0
Q: Confusing differential concept for $f(x)$ and $f(x,y)$.

Aditya AgarwalOkay, so I was reading the concept of "Increment and Differential of a Function". It says: Let us consider a function $y=f(x)$ which has a derivative. The increment of this function $$\Delta y=f(x+\Delta x)-f(x)$$ corresponding to the increment $\Delta x$, has the property that the ratio $\frac{...

0
Q: Probability Problems from hypergeometric to conditional

Tomy(1) In a contest,Harvard sent 3 participants.If there are 10 participants all in all,what is the probability that all three participants are in top 3? (2) In a contest,Harvard sent 3 participants.If there are 10 participants all in all,what is the probability that they will win first,second,thir...

 
0
Q: Should a question with constructive comments be prevented from deletion?

CalimeroTwice lately I posted helpful comments on a couple of questions (same OP for both, second question actually being a followup of the first). Those comments were more like spot-on quick tips than detailed, dignified answers, hence I posted them as such. They indeed guided OP to the solution of hi...

 
0
Q: Finding Area of a triangle inside a semi circle.

nab331I'm familiar with basic high school trig. The answer is 2sin(Theta)cos(Theta) Id appreciate it if someone could give me an explanation.

0
Q: Prove that (0,1) =c ℝ

Jesper MagnussonHow do I prove this knowing that f(x) = tan((pix)2) is a bijection between (0,1) and (0, ∞). We also have a bijection between (-1,1) and (0,1)

Short title. Short question. Prove that (0,1) =c ℝ
0
Q: Random permutation two sets

user44255We have got two sets $A = \{1, \ldots, k \}$ and $B = \{k, \ldots, 2 k - 1\}$. Let $\sigma$ be random permutation of $\{1, \ldots, n\}$, $n \ge 2k - 1$. Let $A_{max} := max\ \sigma(A)$, $B_{min} := min\ \sigma(B)$. What is the probability that $A_{max} = B_{min}$?

0
Q: Noncyclic group has at least 5 subgroup

user152395Let G be a noncyclic group. Show that G has at least 5 subgroup. Anyone help me?

 
10:20 AM
0
Q: New questions in custom list are displayed in the wrong format

AlbireoSometimes when I click on the X more questions bar that appears while displaying a custom question list the new questions are displayed using the wrong format: Note that the format selector in the top right is wrong too: I've actually set to the "expanded", and it's correct for the "old" quest...

0
Q: Making code snippets editable (not necessarily savable) by all users

gmanI'm finding lately I really hate code-snippets. Someone (often myself) posts a code snippet in their question. This is great because I can see the code actually run. But... then I want to try to modify it to see if I can solve the problem. I can't. My only option is to actually click "edit" then ...

 
0
Q: How to give (alternative) proofs of some basic calculus rules using little-o notation?

Tien-Cheng HuangTake two basic differentiation rules (from Baby Rudin) for example: Theorem 5.3 (c). (The quotient rule) Suppose $f$ and $g$ are defined on $[a,b]$ and are differentiable at a point $x \in [a,b]$, and assume that $g(x) \neq 0$, then $(f/g)$ is differentiable at $x$, and $(f/g)'(x) = \frac{g(x)f'...

Tag (proof-writing) should not be the only tag a question has. Please add a tag for a subject area to which the question belongs. (from a bot)Normal Human 29 secs ago
0
Q: Finding the domain and Range of a piece wise Function,

nab331Can someone explain to me how to find the domain and range of a piece wise function using this example? Thanks

0
Q: A limit property on sequences

Sota AntoninoLike the D'Alembert test case for convergence of series I have seen something similar for sequences in my math homework which required proof for this: If we have a sequence $a_{n}$ such that $\space\ lim_{n\rightarrow \infty}|\frac{a_{n+1}}{a_{n}}|<1$ then $\space\ lim_{n\rightarrow \infty}(a_n)...

 
10:37 AM
0
Q: First order logic : how do I solve this problem?

Mithlesh Upadhyay$∀x∃y:R(x,y)∧∀x∀y:(R(x,y)⟹¬R(y,x))∧∀x∀y∀z:(R(x,y)∧R(y,z)⟹R(x,z))∧∀x:¬R(x,x)$ Does it have finite models? Is it satisfiable? If so, give a countable model for it. Genuinely, I'm unable to understand this question. Somewhere, it explained as : broken in $4$ parts, i.e. $A, B, C, D$. It explain...

Title contains problem. Tagged proof-theory. First order logic : how do I solve this problem?
0
Q: Maximum value of $ a^2+b^2+1 $

RigelI'm solving a problem.I have to prove $a^2+b^2+1 <6ab$ to complete the proof. Well, $a$ & $b$ are natural numbers.Is there an elementary way to prove this? I tried plotting the graph fixing the value of $a$.I can't prove it though

0
Q: $\sum_{k=n}^\infty{\frac{1}{k!}} \leq \frac{2}{n!}$

elfeck$$\sum_{k=n}^\infty{\frac{1}{k!}} \leq \frac{2}{n!}$$ Can someone show why this estimate holds true? I tried quite a bit but couldn't reall find a way to approach this. WolframAlpha says it is true but I don't know what the gamma function is. $$ \sum_{k=n}^\infty{\frac{1}{k!}} = \frac{1}{n!} +...

Tag (contest-math) should not be the only tag a question has. Please add a tag for a subject area to which the question belongs. (from a bot)Normal Human 22 secs ago
 
0
Q: Questions display format selector missing from the home page

AlbireoWhen I'm browsing a custom Questions tab I can toggle the questions display format by using the format selector in the top right: However when I'm browsing the home tab the format selector is not shown: However the selector is there: when doing a hard reload it's shown for a moment (FOUC?) ...

 
0
Q: really hard sequences

roniso i failed a math exam, and my professor gave me this question about number sequence for which if i could answer it, i will get a B on my paper. what are the next five sequence of this. 7394, 2263, 8766, 5154, 8021, 6193, ----, ----, ----, ----, ----.?

Short title. Title contains hard. really hard sequences
0
Q: Independence of time series data

OceanI have a time series data with $52$ observations and I would like to check for the independence between observations. The ACF for correlation and covariance of my data look I am aware that $covariance = 0$ does not imply independence, except for Gaussian process. I wonder if I can use ACF to...

Consider adding a tag for a broader subject area to which the question belongs. Some of these tags might fit. (autocomment)Normal Human 21 secs ago
0
Q: Numbers with ear tags

SneftelI have a black-box polynomial function, with $n$ inputs. The only things I know for sure about this polynomial are that (a) it has no constant term, and (b) all coefficients and exponents are integers with magnitude below some $k$. (Exponents are positive, of course.) Let's say I get to evaluate...

0
Q: Rate of change and gradient

Suiz356I'm trying to solve the follow problem: Suppose that we are on the point $P=(1/\sqrt{2},1/2,1/2)$ over $z=\sqrt{1-x^2-y^2},$ $z\geq 0, x^2+y^2<1.$ In which direction we have to move over the surface such as: a) rate of change of z is zero? b) rate of change of z is maximum? My attempt is based ...

 
11:02 AM
0
Q: strike through markdown

Andreas MüllerIt's not working properly when rendering on the site, works in preview though. Found 2 ways striking through in markdown (as seen here): Sample text: <del> #question some text some more text </del> Errors: (#) no longer resulted in heading more importantly: strike through in rendering on ...

 
0
Q: How to apply law of large numbers for this problem

user63416Let $X_1,...,X_n,...$ be independent variable satisfying $P(X_i=0)=P(X_i=1)=\frac{1}{2}$ for all i then denote $Z_i=X_iX_{i+1}$ for all i .I want to show that $lim_{n\to \infty}\frac{Z_1+Z_2+...+Z_n}{n}=\frac{1}{4}$ a.s by using law of large number. I'm new to this area and struck immediately sin...

0
Q: Matrix Norms Question

smithGiven the matrix A $$A:=\left(\begin{array}{cc} 2&1\\ -1&2 \end{array}\right)$$ What is the spectral radius $p(A)$ $||A||_2$ $||A||_\infty$ I got for the spectral radius $p(A)=\sqrt(5)$ for $||A||_2 = \sqrt(5)$ and for $||A||_\infty = 3$ Is this correct?

Words such as question are uninformative in titles. Please edit the title so that it better describes the specifics of your question. Do not hesitate to make it longer or include a formula if needed. More tips here. (from a bot)Normal Human 21 secs ago
0
Q: Deriving $\Delta z=\frac{\partial y}{\partial x}\Delta x+\frac{\partial f}{\partial y}\Delta y+\alpha\sqrt{\Delta x^2+\Delta y^2}$

Aditya AgarwalHow to derive that $$\Delta z=\frac{\partial y}{\partial x}\Delta x+\frac{\partial f}{\partial y}\Delta y+\alpha\sqrt{\Delta x^2+\Delta y^2}$$ Is it a definition? Or there is a proof?

 
11:21 AM
0
Q: Triage: Add explanation what's the reason for having a triage at all

Thomas LandauerRight now, there is almost zero explanation about what the "Triage" is and why it makes sense. On the triage page itself it just says Help identify the quality of questions and We need your help separating questions into categories. Please include (a link to) an explanation like that:...

 
0
Q: $\overline{N_P(H)} = N_\overline{P}(\overline{H})$?

Evgenii.BalaiLet $P$ be a $p$-group. For a subgroup $K$ of P containing $Z(P)$ of G. By $\overline{K}$ we denote the quotient $K/Z(P)$. Let $H$ be a proper subgroup of $P$ containing $Z(P)$. The proof of the first theorem in section 6.1 from the book Abstract Algebra by Dummit and Foote uses the following e...

0
Q: Solvability of Perelman's $\mathcal W$ system.

lanse7ptyHow to show the system have solution ? $R_{ij}$ is ricci tensor, $R$ is scalar curvature. I feel this is complex question, because I have little knowledge about PDE. So, if it is complex, just tell me what I should read ? I try to find answer in Evans' PDE book, but I am not sure. Whether the 11 ...

Tagged pde, differential-equations. Tagged differential-equations but mentions "partial". Solvability of Perelman's $\mathcal W$ system.
0
Q: Nonlinear Schrodinger Equation

OttimistaI have to find equation and starting condition to solve nonlinear Schrodinger Equation with periodic edge condition. This method should control the propagation of fiber optical signal. In details I need to find a valid case as it was already found for the semilinear wave equation (see pages 26-27...

Welcome to Math.SE, Ottimista. This site uses MathJax formatting of formulas. More tips here. (from a bot)Normal Human 21 secs ago
 
0
Q: Sanity check for question closure

PaddyI stumbled across this question today: Ajax success event not working It's 6 years old and has been upvoted highly (I'm not sure why). The question would seem to be off topic as it doesn't contain a minimal, complete and verifiable example of the problem. The title itself is a bit of a red fl...

0
Q: Why was my question on using Ubuntu to trace IPs closed as off-topic?

Paranoid PandaI recently asked this question How to trace the IP of a Tox user?, and although it received upvotes as and was answered, it was still closed as off-topic. But I don't really see why because although it was asking about third-party software, I thought that was allowed, or if it was because it was ...

 
0
Q: How to calculate the height of a cone at particular volume?

BopyThe equation for the volume of a cone is 1/3 x Pi x r^2 x h starting from the base. However, I wanna calculate the height of the cone for a particular volume from the tip of the cone. Could you please help me with a formula? Thanks.

0
Q: Ellipse Standard Form

soupynoodlesHow to convert this equation $$\frac{\left(\frac{a^2b^2x}{a^2y^2+b^2x^2}-p\right)^2}{A^2}+\frac{\left(\frac{a^2b^2y}{a^2y^2+b^2x^2}-q\right)^2}{B^2}=1$$ to the standard form an ellipse? $$\frac{\left(x-p\right)^2}{a^2}+\frac{\left(y-q\right)^2}{b^2}$$ The algebra is really annoying :( Thank...

0
Q: Chain rule with fraction

gboxIn the case of $f(x)=ln(x+\sqrt(1+x^2)$ in the derivative we multiply $f'(x)=\frac{1}{x+\sqrt(1+x^2)}*(1+\frac{2x}{2\sqrt{1+x^2}})$ when the expression multiply the numerator?

Short title. Short question. Chain rule with fraction
0
Q: Bayesian Equilibrium

captaincookWe have to answer the following question, and I can make some progress on the first couple of parts but get stuck in finishing them off. The third part I'm not sure where to start! *Consider the following Bayesian game. Nature chooses the type of player 1 from the set {1, 2, 3, 4} where each typ...

Short title. Bayesian Equilibrium
0
Q: Using Strong Markov Property

An old man in the sea.Let $X_n$ be a DTMC, with transition matrix P and state-space I. Let $Y_m=X_{T_m}$ for $m \in \mathbb{N}$. Define $T_0=\inf\{n\geq0:X_n\in J\subset I\}$ and $T_{m+1}=\inf\{n\geq T_{m}:X_n\in J\subset I\}$. These are stopping times, so we can use the strong markov property when conditioning on th...

0
Q: Bound via using Jacobi sums, $x^3 + y^3$ has solutions mod $p$ for all sufficiently large $p$

Analysis CabalLet $N(x^3 + y^3 = 1)$ be the set of pairs $(x, y) \in \mathbb{F}_p \times \mathbb{F}_p$ such that $x^3 + y^3 = 1$. How do I see that$$\left|N(x^3 + y^3 = 1) - p + 2\right| \le 2\sqrt{p}?$$

0
Q: Application of Baire Theorem.

BarabaraLet $B \subset [0,1]$ be a nowhere dense closed set. $a)$ Show that there exists $s \in [0,1]$ such that for no natural number the point $(s, \frac{s}{n})$ is in the circle $S((0,0),r)$ where $r \in B$. $b)$ Show there is a irrational number satisfying the condition from a). By Baire Theorem, ...

0
Q: sum convergence with Binomial Coefficients

francescoIs Sum[Binomial[(k-1)i,i]((k-1)/k)^((k-2)i))(1/k)^i,{i,1,Infinity}] convergent for any integer k? What is the exact result?

 
12:14 PM
-1
Q: Review: Include explanation about ban (including personal stats)

Thomas LandauerIn the rewiev queues, include explanation about what happens when you fail an audit. I found this answer - if it is accurate, please include it into the review page itself: http://meta.stackoverflow.com/a/278885/1668200 After failing a (bad) audit, I was very upset, since I didn't know what woul...

 
0
Q: Checking if Post Correspondence Problem has a Solution

technoI have the following problem I think that solution is wrong x1=b and y1=b3.They do not match,So how is this solution possible?

Title contains problem. Short question. Checking if Post Correspondence Problem has a Solution
0
Q: $f(m + fn) = fm - n$

IgorWell, similar questions have already been asked. But they still were not identical and the solution methods offered there were not the same. Anyway, i want to solve functional equation $f(m) + f(n)) = f(m) - n$ where $f:\mathbb{Z}\rightarrow \mathbb{Z}$. Somehow i showed that such function doesn'...

A title should not be all-MathJax; having some plain text helps with search and navigation. (autocomment)Normal Human 21 secs ago
0
Q: Show that $Y\sim\Gamma(\frac{1}{2},\frac{1}{2})$

jukka.aaltolet $X\sim N(0,1)$ be a standard normally distributed stochastic variable and let $Y=X^2$. Show that $Y\sim\Gamma(\frac{1}{2},\frac{1}{2})$, ie. $Y\sim\chi^2(1)$.

 
0
Q: What should I do with an answer on my question which has got upvote but not a solution to my question rather It is misleading with that upvote?

Prafulla Kumar SahuSuppose I have a bunty question and some posted an answer and got an upvote, but for my condition the answer is not applicable , but I can notice people are getting confuse with that answer as that is a upvote, It is making people feel, the answer with upvote is really helpful, just need to be im...

0
Q: What to do with answers that are copy and paste of other answers here

MrsEdThis is slightly different from similar questions here. If an answer is a direct copy and paste from another answer here, with a link to the other answer, what should we do about the copied answer? See original here: Calculating the difference between two Java date instances And the copy her...

 
12:34 PM
0
Q: Prove that ∀n ∈ N, n ≥ 1 → H(2n) = H(2n − 1) = n.

Syed NaqiI have this question here that I'm really confused about can someone please tell me how to prove this question the full question is here: I need help with question 7 from this image

Short question. Question contains please. Prove that ∀n ∈ N, n ≥ 1 → H(2n) = H(2n − 1) = n.
0
Q: Combinatorics - problem with Inclusion–exclusion principle

user295008I have a little mathematic problem. I have bar code with 3 types of black (x, y, z) lines and 2 types of white lines (w, v). There are 12 black lines and 11 white lines. And black and white lines alternate like B, W, B, W, B.. 2 black lines are outside. Now should I find how many codes I can mak...

This site uses MathJax formatting of formulas. More tips here. (from a bot)Normal Human 21 secs ago
0
Q: How to find $E(L^q)$ in queue processes?

Marine1Let say we have a Coast Guard center wich have a emergency center hold only one rescuer. accidents comes according to a Poisson process. There is usually 16 casualties on 8 hours. Time passed to healed sufferers is 15 minutes for each of them. Casualties are examined according to an exponential l...

0
Q: Probability questions with survey

Rasheed A school-wide survey revealed that 40% if the students like to eat pizza. If 10 students are randomly selected from the population, what is the probability that 6 likes to eat pizza?

Questions tend to get more attention when they have a tag for a broad area of mathematics relevant to the question. Some of these tags might fit. (autocomment)Normal Human 29 secs ago
0
Q: Let $\mathbf H=\begin{pmatrix}a&b\\0&a\end{pmatrix}$ Show that $e^{\mathbf Ht}=e^{at}\begin{pmatrix}1&bt\\0&1\end{pmatrix}$

bluemoon Let $\mathbf H=\begin{pmatrix}a&b\\0&a\end{pmatrix}$ Show that $e^{\mathbf Ht}=e^{at}\begin{pmatrix}1&bt\\0&1\end{pmatrix}$ I have $$\begin{aligned} &Ht=\begin{pmatrix}at&bt\\0&at\end{pmatrix} \\ & \iff e^{Ht}=e^{\begin{pmatrix}at&bt\\0&at\end{pmatrix}}=e^{at \begin{pmatrix}1&0\\0&1\end{pm...

0
Q: Matrix of LP has not full rank after converison to standard form

user2760995I have coded a reader for MPS files in MatLab, which yields A, b and c as in min c'*x s.t. Ax=b x>=0 The reader transforms the problem from the file to standard form. In the next step I was using a self coded dual and primal simplex to solve this problem. Some small problems from a lectur...

0
Q: Is the inverse relation from $R =_{def} \{ (x,y) | x^2 -1 =y \} \subseteq \mathbb{N} \times \mathbb{R} $ a function?

fragantThe inverse from $R =_{def} \{ (x,y) | x^2 -1 =y \} \subseteq \mathbb{N} \times \mathbb{R} $ is $R^{-1}: y \mapsto (y+1)^{\frac{1}{2}} $ but is this a left-total function?

0
Q: so is my proof correct ?? (about equivalence)

triton21R is a relation of real number . $xRy <-> x+y = 0 $ . is it equivalence relation ? so my answer is no proof : -(Reflexive) let $x = a$ , $ aRa <-> 2a=0$ . so since 2a doesnt hold for every real number a , R isnot reflexive. since R isnt reflexive R isnt equivalence relation. so is my reason...

Title contains ??. Tagged proof-writing. so is my proof correct ?? (about equivalence)
0
Q: Epsilon Delta proof of a limit of a multivariate function

Nimantha PriyamalPlease help me solve this problem. Show that lim (x,y)-->(0,1) ye^x =1 using epsilon delta method.

Short question. Question contains please. Epsilon Delta proof of a limit of a multivariate function
 
1:01 PM
0
Q: An exercise concerning complex numbers

Involutive AutomorphismAssume that $| z + 1 | > 2$. Show that $|z^3 + 1| > 1$. My try was: $$|z^3 + 1| = |z + 1| |z^2 + z + 1| > 2 |z^2 + z + 1| $$ but I'm stuck proving that $|z^2 + z + 1| > \frac 1 2$

Title contains exercise. An exercise concerning complex numbers
0
Q: solve integral with a square root in denominator

Joseph ZhuCan someone help me with this integration question $$\int_0^1 \frac{dx} {x+\sqrt{1-x^2}} $$ I tried substitution with x=sinx and also tries times both denominator and numerator by $$ x- \sqrt {1-x^2} $$ but it becomes even more complicated. Can someone gives me a hint how to solve this questi...

0
Q: What is the logic behind the given optimization problem?

Battle BeastI am following a book which has a part on numerical optimization techniques. In order to elaborate Karush-Kuhn-Tucker theorem, they gave the following example: When the unconstrained solution $x=A^+ b$ does not lie in the feasible region, they apply an iterative approach to find the solution wh...

 
-1
Q: Why run the home-work and exercises tag?

Garvit SharmaMost people on this website, who happen to have some "reputation", don't want to solve homework questions, all they want to do is put questions on hold, because apparently , the questions are "off-topic", and frankly quite stupid, or so I have read. Which coerces me to question, what is the topic...

 
1:17 PM
0
Q: Matlab related question function

Gaussfunction ex2a12(a,x,n) y=zeros(n,1); y(1)=x; for j=1:n y(j)=a*y(j-1)*exp(-y(j-1)); end end I am getting Undefined function 'ex2a12' for input arguments of type 'double'. any ideas how to fix this"?

Words such as question are uninformative in titles. Please edit the title so that it better describes the specifics of your question. Do not hesitate to make it longer or include a formula if needed. More tips here. (from a bot)Normal Human 21 secs ago
0
Q: Evaluate the stability of a fixed point using direct linearisation.

Callum(b) Can anyone shed some light on the method for solving this problem?

0
Q: Probability exercise about SLLN

Hu ju yuanThe question: Let {$X_n$} be i.i.d random variables. $EX_1=0$. Then $\sum_{i=1}^{n}X_i\over n$ converges almost surely to zero. I know that when the sequence $\{X_n\}$ satisfies $\sum_{i=1}^{\infty}{Var(X_i)\over i^2} \lt \infty$, the conclusion holds. So I try to cut $X_n$ to make it satisfy th...

Title contains exercise. Probability exercise about SLLN
0
Q: Composition of various combinations of one one,many one,onto and into functions.

user984325Let $f:R\to R$ be an one-one function,$g:R\to R$ be a many one function,$h:R\to R$ be an onto function and $l:R\to R$ be an into function. $(1)$I know that one one function composition one one function is a one one function. $(2)$What is one one function composition many one function. $(3)$What ...

Consider adding a tag for a broader subject area to which the question belongs. Some of these tags might fit. (from a bot)Normal Human 21 secs ago
0
Q: Order of $ab$ and inverses

Rishabh SareenLet $a$ and $b$ be elements of a group, with $a^2=e, b^6=e$ and $ab=b^4a.$ Find the order of $ab$ and express the inverse in each of the terms $a^mb^n$ and $b^ma^n.$ Just want to cross check my solution. Would be very grateful for the complete solution.

0
Q: What's wrong with my proof on "Any countably incomplete ultraproduct of a collection of models is $\aleph_1$-saturated

Amir.H KianiI'm using this article for the proof. I thought some parts are extra and tried to make a new shorter proof. Here goes: Let $\Delta(x)$ be a set of formulas (with one free-variable $x$) in the language $\mathcal L$. It suffices to show that if each finite subset of $\Delta(x)$ is realized in the...

0
Q: Find general inverse of $A$

ghazaleFind general inverse of $A$ where $ A = \left( \begin{array}{ccccc} 1 & -1 & 0 & 0 \\ 0 & 3& 2&1 \\ -1 & 1 & 1&1\\ 2&1&0&4\\ 0&1&0&1 \end{array} \right) $ can you help me?

0
Q: What is the value of $\Sigma(f_ix_i-\bar x)$?

Aditya AgarwalLet $f_i, \ i=1,2,...n$ be the frequencies of class intervals. Let $x_i, \ i=1,2,...n$ be the class marks. And $\bar x$ is the mean. What is the value of $\Sigma(f_ix_i-\bar x)$? I got this question in a test. I was confused, whether, a solution even existed or not? Because inserting real values...

0
Q: Null Set Symbol

MattMy professor pitched me a problem like so, 2∅(25) mod 25 ≡ ? The answer is 1. However, after much browsing of the internet, I couldn't find anything to justify this answer. I was wondering exactly how she came to this conclusion.

Short title. Null Set Symbol
0
Q: If f(a) < f(p) and f(p) > f(b) then there is a d such that f'(d)=0 proof

user295542If $f: [a,b] → R$ is a continuous function which is differentiable on $(a,b)$ If $f(a) < f(p)$ and $f(p) > f(b)$ for some $p ∈ (a, b)$. Show that there exists $d ∈ (a, b)$ such that $f'(d) = 0$ I know I need to use Rolle's Theorem and the intermediate value theorem to do this, however I am unsu...

 
1:46 PM
0
Q: Change the question title if real issue is different

GNKeshavaCheck this question, for example. Here, the question title is C program stopped working after use scanf. But the real issue has nothing to do with scanf. Instead, it was because of wrong syntax used for printf. Now, after OP accepting the actual solution, is it OK to change the question title ...

 
0
Q: Finding CDF of a distribution

mkropkowskiRandom variable X has uniform distribution on a line segment $[-3, 3]$, $Y=X^2$ Find cdf of the variable Y. $$ Fy= \begin{cases} \sqrt{t}/3&for &t\in[0,9] \\ 1 &for & t\ge9 \end{cases} $$ I mean why do we change those "boundaries" why we have distribution on $[-3,3]$ and then we change it into $[...

0
Q: relation betwen two random variables related by indicator function

soumayaif any one can help me please if we consifer two continuous random variable X and Y ; we suppose that enter image description here is that any relation betwwen the Expected value of X and Y or ather mathematical expressions that related X and Y . thank you

0
Q: How do I simplify this Boolean expression?

Stephen Wisniewski(A’B’C) + (A’BC) + (ABC’) I'm struggling with this concept..

0
Q: Probability conditional

TomyThe cafeteria offers discount to students who opt to buy two meals.As part of the plan,the students may chooses exactly two among the meals A,B,and C.If they opt to buy only one meal,then they cannot avail the discount.The proportions of the students who chose meals A,B ,and C are 1/6,1/3,and 5/1...

 
00:00 - 14:0014:00 - 00:00

« first day (31 days earlier)      last day (543 days later) »